Proof by Induction - New to proofsDominoes and induction, or how does induction work?Proving that...

How to prepare vegetables for a sandwich that can last for several days in a fridge?

If all harmonics are generated by plucking, how does a guitar string produce a pure frequency sound?

raspberry pi change directory (cd) command not working with USB drive

Is 45 min enough time to catch my next flight

What's the purpose of these copper coils with resitors inside them in A Yamaha RX-V396RDS amplifier?

What to do when being responsible for data protection in your lab, yet advice is ignored?

Is the theory of the category of topological spaces computable?

When does coming up with an idea constitute sufficient contribution for authorship?

4 Spheres all touching each other??

Obtaining a matrix of complex values from associations giving the real and imaginary parts of each element?

How do we edit a novel that's written by several people?

Meaning of すきっとした

Why do neural networks need so many training examples to perform?

How to add multiple differently colored borders around a node?

What was the population of late Pre-Islamic Arabia and the population of Arabic speakers before Islam?

How to properly claim credit for peer review?

Word to be used for "standing with your toes pointing out"

Can a person refuse a presidential pardon?

Meth dealer reference in Family Guy

Can the Count of Monte Cristo's calculation of poison dosage be explained?

What is the meaning of "pick up" in this sentence?

What is Crew Dragon approaching in this picture?

Dilemma of explaining to interviewer that he is the reason for declining second interview

Auto Insert date into Notepad



Proof by Induction - New to proofs


Dominoes and induction, or how does induction work?Proving that $frac{phi^{400}+1}{phi^{200}}$ is an integer.Generating induction proofs from graphs/integralsMathematical induction proof; $g_k=3g_{k-1} - 2g_{k-2}$induction proof for kleene starTips on constructing a proof by induction.Induction proofs for subsets of integersproof using the mathematical inductionProof by Induction involving divisibilityInduction Proof:Inequality proof by induction(?)Proof by induction?













2












$begingroup$


totally new to proofs and found this challenge problem and struggling a bit. Any help would be appreciated!



There are some real numbers $x$ such that $x+frac{1}{x}$ is an
integer. For example, $2+sqrt{3}+frac{1}{2+sqrt{3}}=4$,
$1+frac{1}{1}=2$, and $2sqrt{6}-5+frac{1}{2sqrt{6}-5}=-10$.



Prove for all $xinmathbb{R}$ that if $x+frac{1}{x}$ is an integer,
then $x^n +frac{1}{x^n}$ also is an integer for all $ninmathbb{N}$.










share|cite|improve this question







New contributor




Robin is a new contributor to this site. Take care in asking for clarification, commenting, and answering.
Check out our Code of Conduct.







$endgroup$












  • $begingroup$
    I recommend this answer as a good start
    $endgroup$
    – Ross Millikan
    1 hour ago










  • $begingroup$
    math.stackexchange.com/questions/936479/…
    $endgroup$
    – lab bhattacharjee
    1 hour ago
















2












$begingroup$


totally new to proofs and found this challenge problem and struggling a bit. Any help would be appreciated!



There are some real numbers $x$ such that $x+frac{1}{x}$ is an
integer. For example, $2+sqrt{3}+frac{1}{2+sqrt{3}}=4$,
$1+frac{1}{1}=2$, and $2sqrt{6}-5+frac{1}{2sqrt{6}-5}=-10$.



Prove for all $xinmathbb{R}$ that if $x+frac{1}{x}$ is an integer,
then $x^n +frac{1}{x^n}$ also is an integer for all $ninmathbb{N}$.










share|cite|improve this question







New contributor




Robin is a new contributor to this site. Take care in asking for clarification, commenting, and answering.
Check out our Code of Conduct.







$endgroup$












  • $begingroup$
    I recommend this answer as a good start
    $endgroup$
    – Ross Millikan
    1 hour ago










  • $begingroup$
    math.stackexchange.com/questions/936479/…
    $endgroup$
    – lab bhattacharjee
    1 hour ago














2












2








2





$begingroup$


totally new to proofs and found this challenge problem and struggling a bit. Any help would be appreciated!



There are some real numbers $x$ such that $x+frac{1}{x}$ is an
integer. For example, $2+sqrt{3}+frac{1}{2+sqrt{3}}=4$,
$1+frac{1}{1}=2$, and $2sqrt{6}-5+frac{1}{2sqrt{6}-5}=-10$.



Prove for all $xinmathbb{R}$ that if $x+frac{1}{x}$ is an integer,
then $x^n +frac{1}{x^n}$ also is an integer for all $ninmathbb{N}$.










share|cite|improve this question







New contributor




Robin is a new contributor to this site. Take care in asking for clarification, commenting, and answering.
Check out our Code of Conduct.







$endgroup$




totally new to proofs and found this challenge problem and struggling a bit. Any help would be appreciated!



There are some real numbers $x$ such that $x+frac{1}{x}$ is an
integer. For example, $2+sqrt{3}+frac{1}{2+sqrt{3}}=4$,
$1+frac{1}{1}=2$, and $2sqrt{6}-5+frac{1}{2sqrt{6}-5}=-10$.



Prove for all $xinmathbb{R}$ that if $x+frac{1}{x}$ is an integer,
then $x^n +frac{1}{x^n}$ also is an integer for all $ninmathbb{N}$.







induction






share|cite|improve this question







New contributor




Robin is a new contributor to this site. Take care in asking for clarification, commenting, and answering.
Check out our Code of Conduct.











share|cite|improve this question







New contributor




Robin is a new contributor to this site. Take care in asking for clarification, commenting, and answering.
Check out our Code of Conduct.









share|cite|improve this question




share|cite|improve this question






New contributor




Robin is a new contributor to this site. Take care in asking for clarification, commenting, and answering.
Check out our Code of Conduct.









asked 1 hour ago









RobinRobin

261




261




New contributor




Robin is a new contributor to this site. Take care in asking for clarification, commenting, and answering.
Check out our Code of Conduct.





New contributor





Robin is a new contributor to this site. Take care in asking for clarification, commenting, and answering.
Check out our Code of Conduct.






Robin is a new contributor to this site. Take care in asking for clarification, commenting, and answering.
Check out our Code of Conduct.












  • $begingroup$
    I recommend this answer as a good start
    $endgroup$
    – Ross Millikan
    1 hour ago










  • $begingroup$
    math.stackexchange.com/questions/936479/…
    $endgroup$
    – lab bhattacharjee
    1 hour ago


















  • $begingroup$
    I recommend this answer as a good start
    $endgroup$
    – Ross Millikan
    1 hour ago










  • $begingroup$
    math.stackexchange.com/questions/936479/…
    $endgroup$
    – lab bhattacharjee
    1 hour ago
















$begingroup$
I recommend this answer as a good start
$endgroup$
– Ross Millikan
1 hour ago




$begingroup$
I recommend this answer as a good start
$endgroup$
– Ross Millikan
1 hour ago












$begingroup$
math.stackexchange.com/questions/936479/…
$endgroup$
– lab bhattacharjee
1 hour ago




$begingroup$
math.stackexchange.com/questions/936479/…
$endgroup$
– lab bhattacharjee
1 hour ago










1 Answer
1






active

oldest

votes


















7












$begingroup$

HINT: Note that for $ngeq1$ you have
$$left(x^n+frac{1}{x^n}right)left(x+frac{1}{x}right)=left(x^{n+1}+frac{1}{x^{n+1}}right)+left(x^{n-1}+frac{1}{x^{n-1}}right).$$



For more details, hover over the the block below:




The equation above can be rewritten to get
$$x^{n+1}+frac{1}{x^{n+1}}=left(x^n+frac{1}{x^n}right)left(x+frac{1}{x}right)-left(x^{n-1}+frac{1}{x^{n-1}}right).$$
If the three terms in parentheses on the right hand side are integers, then so is the left hand side. Now to use induction, all you need is that $x^n+frac{1}{x^n}$ is an integer for $n=0$ and $n=1$.







share|cite|improve this answer









$endgroup$













    Your Answer





    StackExchange.ifUsing("editor", function () {
    return StackExchange.using("mathjaxEditing", function () {
    StackExchange.MarkdownEditor.creationCallbacks.add(function (editor, postfix) {
    StackExchange.mathjaxEditing.prepareWmdForMathJax(editor, postfix, [["$", "$"], ["\\(","\\)"]]);
    });
    });
    }, "mathjax-editing");

    StackExchange.ready(function() {
    var channelOptions = {
    tags: "".split(" "),
    id: "69"
    };
    initTagRenderer("".split(" "), "".split(" "), channelOptions);

    StackExchange.using("externalEditor", function() {
    // Have to fire editor after snippets, if snippets enabled
    if (StackExchange.settings.snippets.snippetsEnabled) {
    StackExchange.using("snippets", function() {
    createEditor();
    });
    }
    else {
    createEditor();
    }
    });

    function createEditor() {
    StackExchange.prepareEditor({
    heartbeatType: 'answer',
    autoActivateHeartbeat: false,
    convertImagesToLinks: true,
    noModals: true,
    showLowRepImageUploadWarning: true,
    reputationToPostImages: 10,
    bindNavPrevention: true,
    postfix: "",
    imageUploader: {
    brandingHtml: "Powered by u003ca class="icon-imgur-white" href="https://imgur.com/"u003eu003c/au003e",
    contentPolicyHtml: "User contributions licensed under u003ca href="https://creativecommons.org/licenses/by-sa/3.0/"u003ecc by-sa 3.0 with attribution requiredu003c/au003e u003ca href="https://stackoverflow.com/legal/content-policy"u003e(content policy)u003c/au003e",
    allowUrls: true
    },
    noCode: true, onDemand: true,
    discardSelector: ".discard-answer"
    ,immediatelyShowMarkdownHelp:true
    });


    }
    });






    Robin is a new contributor. Be nice, and check out our Code of Conduct.










    draft saved

    draft discarded


















    StackExchange.ready(
    function () {
    StackExchange.openid.initPostLogin('.new-post-login', 'https%3a%2f%2fmath.stackexchange.com%2fquestions%2f3134247%2fproof-by-induction-new-to-proofs%23new-answer', 'question_page');
    }
    );

    Post as a guest















    Required, but never shown

























    1 Answer
    1






    active

    oldest

    votes








    1 Answer
    1






    active

    oldest

    votes









    active

    oldest

    votes






    active

    oldest

    votes









    7












    $begingroup$

    HINT: Note that for $ngeq1$ you have
    $$left(x^n+frac{1}{x^n}right)left(x+frac{1}{x}right)=left(x^{n+1}+frac{1}{x^{n+1}}right)+left(x^{n-1}+frac{1}{x^{n-1}}right).$$



    For more details, hover over the the block below:




    The equation above can be rewritten to get
    $$x^{n+1}+frac{1}{x^{n+1}}=left(x^n+frac{1}{x^n}right)left(x+frac{1}{x}right)-left(x^{n-1}+frac{1}{x^{n-1}}right).$$
    If the three terms in parentheses on the right hand side are integers, then so is the left hand side. Now to use induction, all you need is that $x^n+frac{1}{x^n}$ is an integer for $n=0$ and $n=1$.







    share|cite|improve this answer









    $endgroup$


















      7












      $begingroup$

      HINT: Note that for $ngeq1$ you have
      $$left(x^n+frac{1}{x^n}right)left(x+frac{1}{x}right)=left(x^{n+1}+frac{1}{x^{n+1}}right)+left(x^{n-1}+frac{1}{x^{n-1}}right).$$



      For more details, hover over the the block below:




      The equation above can be rewritten to get
      $$x^{n+1}+frac{1}{x^{n+1}}=left(x^n+frac{1}{x^n}right)left(x+frac{1}{x}right)-left(x^{n-1}+frac{1}{x^{n-1}}right).$$
      If the three terms in parentheses on the right hand side are integers, then so is the left hand side. Now to use induction, all you need is that $x^n+frac{1}{x^n}$ is an integer for $n=0$ and $n=1$.







      share|cite|improve this answer









      $endgroup$
















        7












        7








        7





        $begingroup$

        HINT: Note that for $ngeq1$ you have
        $$left(x^n+frac{1}{x^n}right)left(x+frac{1}{x}right)=left(x^{n+1}+frac{1}{x^{n+1}}right)+left(x^{n-1}+frac{1}{x^{n-1}}right).$$



        For more details, hover over the the block below:




        The equation above can be rewritten to get
        $$x^{n+1}+frac{1}{x^{n+1}}=left(x^n+frac{1}{x^n}right)left(x+frac{1}{x}right)-left(x^{n-1}+frac{1}{x^{n-1}}right).$$
        If the three terms in parentheses on the right hand side are integers, then so is the left hand side. Now to use induction, all you need is that $x^n+frac{1}{x^n}$ is an integer for $n=0$ and $n=1$.







        share|cite|improve this answer









        $endgroup$



        HINT: Note that for $ngeq1$ you have
        $$left(x^n+frac{1}{x^n}right)left(x+frac{1}{x}right)=left(x^{n+1}+frac{1}{x^{n+1}}right)+left(x^{n-1}+frac{1}{x^{n-1}}right).$$



        For more details, hover over the the block below:




        The equation above can be rewritten to get
        $$x^{n+1}+frac{1}{x^{n+1}}=left(x^n+frac{1}{x^n}right)left(x+frac{1}{x}right)-left(x^{n-1}+frac{1}{x^{n-1}}right).$$
        If the three terms in parentheses on the right hand side are integers, then so is the left hand side. Now to use induction, all you need is that $x^n+frac{1}{x^n}$ is an integer for $n=0$ and $n=1$.








        share|cite|improve this answer












        share|cite|improve this answer



        share|cite|improve this answer










        answered 1 hour ago









        ServaesServaes

        26.5k33997




        26.5k33997






















            Robin is a new contributor. Be nice, and check out our Code of Conduct.










            draft saved

            draft discarded


















            Robin is a new contributor. Be nice, and check out our Code of Conduct.













            Robin is a new contributor. Be nice, and check out our Code of Conduct.












            Robin is a new contributor. Be nice, and check out our Code of Conduct.
















            Thanks for contributing an answer to Mathematics Stack Exchange!


            • Please be sure to answer the question. Provide details and share your research!

            But avoid



            • Asking for help, clarification, or responding to other answers.

            • Making statements based on opinion; back them up with references or personal experience.


            Use MathJax to format equations. MathJax reference.


            To learn more, see our tips on writing great answers.




            draft saved


            draft discarded














            StackExchange.ready(
            function () {
            StackExchange.openid.initPostLogin('.new-post-login', 'https%3a%2f%2fmath.stackexchange.com%2fquestions%2f3134247%2fproof-by-induction-new-to-proofs%23new-answer', 'question_page');
            }
            );

            Post as a guest















            Required, but never shown





















































            Required, but never shown














            Required, but never shown












            Required, but never shown







            Required, but never shown

































            Required, but never shown














            Required, but never shown












            Required, but never shown







            Required, but never shown







            Popular posts from this blog

            Why do type traits not work with types in namespace scope?What are POD types in C++?Why can templates only be...

            Will tsunami waves travel forever if there was no land?Why do tsunami waves begin with the water flowing away...

            Should I use Docker or LXD?How to cache (more) data on SSD/RAM to avoid spin up?Unable to get Windows File...